Find the mean of the data in the dot plot below.

\text{ g} gstart text, space, g, end text

Answers

Answer 1

Answer: 40.5

Step-by-step explanation: If it help please thanks me!

Answer 2

Answer:

???picture please

Step-by-step explanation:


Related Questions



2. What is the center of dilation and the scale factor of the dilation of ABCD to EFGH?

Answers

Answer:

The center of dilation of ABCD to EFGH is (0, 0), and the scale factor is 4/3

Pavi has a credit line of $8,000 on his credit card. Review the summary of his credit card statement. How much credit does Pavi currently have available on this card? Summary Previous Balance Payments/Credits New Purchases Finance Charge New Balance $2,500.00 $1,500.00 $3,000.00 $7.50 $4,007.50

Answers

Answer:

$3,992.50

Step-by-step explanation:

According to the problem, computation of the given data are as follows,

Credit line on credit card = $8,000

Balance Payment = $2,500

Credit = $1,500

New Purchases = $3,000

Finance Charges = $7.50

New Balance = $4,007.50

So we can calculate the  available credit on card by using following formula,

Available credit on card = Credit line on credit card -New balance

By putting the value, we get

Available credit on card = $8,000 - $4,007.50

= $3,992.50

The volume of a square pyramid with a base length of 12 cm and a height of 10 cm is ___ cubic cm

Answers

Answer:

480 cm³

Step-by-step explanation:

Formula for Pyramid volume = (l*w*x) / 3

we do 12 x 12 x 10 first =

12 x 12 = 144

144 x 10 = 1440

Then divide by 3

1440 / 3 = 480

units = cm³

Answer = 480 cm³

If my answer is incorrect, pls correct me!

If you like my answer and explanation, mark me as brainliest!

-Chetan K

HELP ME ASAP!!! On a scale used on a map, 3/4 inch represents 8 feet. On the map, what is the length of a room with an actual length of 24 feet?



A
4 in.

B[tex]1 \frac{3}{4} [/tex]

C
[tex]4 \ \frac{1}{2} [/tex]

D
[tex]2 \ \frac{1}{4} [/tex]

Answers

Answer:

D. 2 1/4

Step-by-step explanation:

3/4 inch = 8 feet

24 divided by 8 = 3

(meaning you would multiply eight three times in order to get to 24)

So....

3/4 x 3 = 2 1/4

Answer: 2 1/4

Hope this helps :)

Answer: D - 2 1/4”

Step-by-step explanation:

8 feet goes into 24 feet 3 times

Eg. 24/8 = 3

If 3/4” = 8 feet, then 3 x 3/4” = 24 feet

3 x 3/4” = 2 1/4”

Suppose fx) = x? What is the graph of g(x)= x(x)?

Answers

Answer:

1/2x^2

Step-by-step explanation:

f(x)=x^2 so g(x)=1/2x^2

if g(x)=x(x) then the answer is 1/2x^2

use desmos to help u select the correct graph

. Solve for x. a) 10 - 1/4x - 7 b) 3 (x + 8 ) = 21

Answers

a) 10 - 1/4x = 7

   1/4x = 7 + 10

           = 17

   x = 17/ (1/4)

b) 3 (x + 8 ) = 21

   x + 8 = 21/3

            = 7

   x = 7 - 8

      = -1

What percent of the values are between 10 and 30? Round to the nearest tenth of a percent if necessary.

Answers

Answer: 33.3%

Step-by-step explanation: I did this on saavas. also for part b the answer is 32.5.

Robert made 7 loaves of bread. Each loaf of bread uses of a cup of flour. How many cups of flour did Robert use to make 7 loaves of bread?

Answers

Answer:

7 cups of flower.

Step-by-step explanation:

He made 7 loaves and each loaf calls for 1 cup of flour. 7 times 1=7...I feel like this is so easy.

lets find the answer

Answers

Answer:

-6,3....................

Answer:

(-4,3)

Step-by-step explanation:

There are 48 cups in a box of paper cups. How many cups are in 70 boxes of cups?

Answers

Answer:

3360

bc

48 cups x 70 boxes = 3360

Step-by-step explanation:

Use the Law of Cosines to write an expression equivalent to c. Drag the expressions to show what the values of the variables should be in the radical.

Answers

Answer:

[tex] \boxed{\begin{array}{ccc} m = {a}^{2} \\ \hline n = {b}^{2} \\ \hline p = 2ab \\ \hline q = \cos(C) \end{array}}[/tex]

Step-by-step explanation:

recall the formula for law of cosine

[tex] \rm \displaystyle {c}^{} =\sqrt{ {a}^{2} + {b}^{2} - 2ab \cos(C) }[/tex]

thus we acquire

m=a²n=b²p=2abq=Cos(C)

Answer:

Solution given:

m=a²

n=b²

p=2ab

q=Cos(C)

law of cosine

[tex] c = \sqrt{a²+b²−2abcos(C)} [/tex]

Find the missing value to the nearest whole number. sin17∘=x/7

Answers

Answer:

2

Step-by-step explanation:

sin 17° = x/7

(sin 17°) 7 = (x/7)7

x = sin 17° (7)

x = 0.292371704722737 (7)

x = 2.046601933059157

x ≈ 2

what is the equation for the line?

Answers

Answer:

y=6/5x+8/5

Step-by-step explanation:

find the slope (6/5) you can find this by making a slope triangle between the 2 points.

now, plug in a point to y=6/5x+b

-2=6/5(-3)+b

-2=-18/5+b

-10/5=-18/5+b

b=8/5 (y-int)

so y=6/5x+8/5

What is the mode of the following set of numbers: 3, 5, 6, 9, 3, 3, 6
A. 5
B. 3
C. 35

Answers

B. 3
3 is the mode because it occurs the most.
The answer is A. 6
Hope this helps.
The mode is difference between the lowest and highest number so the difference between 3 to get to 9 is 6. 6+3 =9.

There are 1,024 players in a tennis tournament. In each round, half the players are eliminated. Which function can be used to find the number of players remaining in the tournament at the end of x rounds?

Answers

Answer:

1,024 ÷ by how many number of rounds= x

Answer:

1,024(0.50)^x That is the answer......................................

expand and simplify
3(x+1)+2(x+2)​

Answers

Answer:

5x+7

Step-by-step explanation:

3x+3+2x+4

3x+2x+3+4

5x+7

plz mark as brainliest

The school that Stefan goes to is selling tickets to a choral performance. On the first day of ticket sales the school sold 3 senior citizen tickets and 1 child ticket for a total of $38. The school took in $52 on the second day by selling 3 senior citizen tickets and 2 child tickets. Find the price of a senior citizen ticket and the price of a child ticket. ​

Answers

Answer:

3s + c = 38 3s + 2c = 52

Step-by-step explanation:

Answer: Senior Citizen Ticket = $8, Child Ticket = $12

Step-by-step explanation:

Let's use S for senior tickets, C for child tickets.

3S + 1C = 38

3S + 2C = 52

Find C in terms of S with the first equation:

C = 38 - 3S

Now substitute the above equation for C in our second equation to find S.

3S + 2(38 - 3S) = 52

3S + 76 - 6S = 52

-3S = -24

S = 8

Now plug this value for S back into the first equation:

3(8) + C = 38

C = 12

So S = $8 and C = $12

Which answer represents the series in sigma notation? −7+(−2)+3+8+13 ∑j=15(5j−12) ∑j=15(−8j+1) ∑j=15−5(15)j−1 ∑j=155j−1

Answers

Answer:

∑j=15(5j−12)

Step-by-step explanation:

For j = 1 up to j = 5

j = 1

(5j - 12) = (5(1) - 12) = 5 - 12 = - 7

j = 2

(5j - 12) = (5(2) - 12) = 10 - 12 = - 2

j = 3

(5j - 12) = (5(3) - 12) = 15 - 12 = 3

j = 4

(5j - 12) = (5(4) - 12) = 20 - 12 = 8

j = 5

(5j - 12) = (5(5) - 12) = 25 - 12 = 13

(-7 + (-2) + 3 + 8 + 13)

Answer:

(5j-12)

Step-by-step explanation:

Other person was right, took the test using the answer they gave.

4x-2(x-5)=-8+4x+12 answer plz help #lovedafans

Answers

Answer:

x = 3

Step-by-step explanation:

Answer:

The answer is x = 3

can ace and expert help me

in which number does 7 resent 10 times the value that it respesnts in 167 300​

Answers

Answer:

D

Step-by-step explanation:

repeat

Question :-

In which number does 7 represent 10 times the value that it represents in 167 300.

____________________

Solution :-

Given Information :-

Number ➢ 167 300

To Find :-

In which number does 7 represent 10 times the value that it represents in 167 300.

215,007392,070450,700572,000791,000

Calculation :-

In the given number, we can observe that, the face value of digit 7 is on 'Thousands place'. If multiply 7, 10 times, the face value of digit 7 will shift to it's Left Side by 1 digit, since, this condition is seen in the 4th option provided, hence, option 4. 572,000 is the correct answer.

____________________

Final Answer :-

Option 4. 572,000 is the correct answer

___________________

find the LCM 72 ,216, 270

Answers

Answer:

1080

Step-by-step explanation:

72 = 2 × 2 × 2 × 3 × 3

216 = 2 × 2 × 2 × 3 × 3 × 3

270 = 2 × 3 × 3 × 3 × 5

LCM(72, 216, 270)

= 2 × 2 × 2 × 3 × 3 × 3 × 5

= 1080

Answer:  1080

==========================================

Explanation:

Write out the prime factorization of each value. A factor tree may help.

72 = 2^3*3^2216 = 2^3*3^3270 = 2*3^2*5

The unique primes that show up are: 2, 3, 5

For the prime factor 2, the highest exponent is 3. So 2^3 is a factor of the LCM.For the prime factor 3, the highest exponent is 3. So 3^3 is a factor of the LCM.For the prime factor 5, the highest exponent is 1. So 5^1 is a factor of the LCM

The LCM is 2^3*3^3*5^1 = 8*27*5 = 1080

please help me whit this (if you answer pls show the work)

Answers

Answer:

24 ft

Step-by-step explanation:

Use proportions! The old tent has sides of 10ft, and a base of 15ft. The new tent has sides of 16ft and a base of ____ ft.

So, 10/15 = 16/___. Cross multiple and you get 24ft!

Praise put some money into an account paying 4.5% compounded continuously for 10 year
She now has $3567.91 in the account. How much money did she start the account with?

Answers

Answer: [tex]\$2297.47[/tex]

Step-by-step explanation:

Given

Rate of interest [tex]r=4.5\%[/tex]

time period [tex]t=10\ year[/tex]

Amount [tex]A=\$3567.91[/tex]

Amount in compound interest is given by

[tex]\Rightarrow A=P[1+r\%]^t[/tex]

where, [tex]P=\text{Principal}[/tex]

Insert the values

[tex]\Rightarrow 3567.91=P[1+0.045]^{10}\\\\\Rightarrow P=\dfrac{3567.91}{1.5529}\\\\\Rightarrow P=\$2297.47[/tex]

So, Praise starts with [tex]\$2297.47[/tex]

Carmella likes to hike for more than 4 hours but less than 6 hours. She hikes at a speed of 2 miles per hour.
If m is the number of miles Carmella likes to hike, which inequality is true?
A. 0.33 B 2.0 C 4.0 D 6.0 E 8.0

Answers

Answer:

Guys the answer is E. I just took the test.

Step-by-step explanation:

Short answer: Look at the inequalities and plug in either 10 or 8 for m and then you're all set :)

Long Answer:So what you should do first is calculate how long she hikes for and how many mph. It says that she likes the hike for more than 4 hours but less than six. She also hikes at least 2 mph. So if you were ever in first grade you would know that she hikes for at least 5 hours. Now 5 x 2 is 10. Now you can either plug 8.0 into the inequalities or 10 but either way its going to be more than whatever number is first in the inequality but less than 12.

Answer:

E.

Step-by-step explanation:

pay:11.71 per hour
hours:11 hours per week​

Answers

Week = 128.81
Month = 515.24
Year = 6182.88
I believe this is all correct :)

Answer:

$128.81 per week

$515.24 per month

$6182.88 per year

Step-by-step explanation:

[tex]\frac{11.71}{y} :\frac{1}{11}[/tex]

y = 128.81

[tex]\frac{128.81}{y} :\frac{1}{4}[/tex]

y = 515.24

[tex]\frac{515.24}{y}: \frac{1}{12}[/tex]

y = 6182.88

Events without replacement are independent events.

false

true

Answers

Answer:

True

Step-by-step explanation:

When sampling is done with replacement, then events are considered to be independent, meaning the result of the first pick will not change the probabilities for the second pick. Without replacement: When sampling is done without replacement, each member of a population may be chosen only once.

Expand to write an equivalent expression. −5(7−2)

Answers

-35+10
If correct please mark as Brainly!

someone please help!

Answers

Answer: 62cm^2

Step-by-step explanation: 70-8=62

Determine side x. Round to the nearest unit. 20 9 12 5

Answers

Answer:

12

Step-by-step explanation:

First, you will have to use trigonometry. Since the side of the triangleis the opposite of the angle 53 and the hypotenuse, you will use sin.

Sin 53 degrees = opposite side / hypotenuse side.

The opposite side is x and the hypotenuse side equals 15.

sin53= x/15

15 ( sin 53 ) =x

x= 11.97953

Round

x=12

PLEASE HELP MATH I DON'T GET IT

Answers

I think that y=something on the graph and x= something on the graph
Other Questions
A substance has a pH of 4. Is it an acid or a base? Tim earns $52.50 for 5 hours of dog walking. He earns the same amountper hour each time he dog walks. How many hours does Tim need to dogwalk to earn $168? find the coordinates of point p along the direct line segment ST so that SP to PT is 1 to 3. S= (-6,7) and T=(9,25) [tex] \sqrt[2]{a} {}^{2} [/tex] Factor 32x15 . If the expression cannot be factored, write cannot be factored. Help please someone please what is the answer Present Subjunctive - Adjective Clauses: Practice 1 Instrucciones Cada una de las oraciones contiene una clusula adjetival pero las oraciones no estn en el orden correcto. Cambia el orden de las palabras para que la oracin tenga sentido. /133 1. gerente busca sepa El a hablar que espaol. alguien 2. sea que mis sueldo pagar para suficiente un Necesito deudas. 3. alcanzables. sean fijar que deben estudiantes Los metas 4. accin. que Quiero pelcula ver una tenga mucha 5. dedicados. compaa empleados existe no que tenga No una exitosa 6. busca se una universidad que Juan en ciencias. especialice 7. No aos. menos haga que conozco a en de 2 la maestra nadie 8. buscan que que jefes tengan mejores empleados nuevos a Los empleados pasados. habilidades 9. Marta el que ahora. aun no ms quiere cargos trabajo cumplir un difciles que tiene que tenga de 10.en que tienes No mejor hay ahora. paguen trabajo que ciudad que el esta trabajos 11 octo algunas comestre fciles on matricularnos What do you notice about the narrative voice and viewpoint in the novel To Kill A Mockingbird? can someone help me with this question? Measure the volume of the gas in the syringe.Estimate it to the nearest 0.5 mL. mL30.5Intro32.537.540.0 Jen is planning a trip to France. The exchange rate is 3 Euros for $4. How many Euros will she get if she exchanged $28? DO NOT FORGET THE UNITS! What is the shape of the cross section of the triangular prism in each situation? Drag and drop the answer into the box to match each situation. What is the range of 39,41,5,43,26,7,43,24,41 The answer is the first or last option but I dont know which one Points A, B, C, and D are connected to form square ABCD. If square EFGH is similar to square ABCD, what are the coordinates of G? A) (4, 4) B) (4, 5) C) (5, 4) D) (5, 5) In most natural populations rapid exponential growth is unsustainable. As populationsincrease, environmental resistance causes the growth rate to slow down, until carryingcapacity is reached. Brainstorm several factors that could be considered as environmentalresistance. Which of the following statements about the economies of the United Kingdom, Germany, and Russia is false?A. All three countries are considered mixed economies.B. All three countries have service industries.C. Russia has more government regulations in its economic system than the other two countries.D. Germany and the United Kingdom have developing economies, while Russia has a highlydeveloped economy.Please select the best answer from the choices provided BDMark this and returnSave and ExitNextSubmitO Which of the following is NOT a strategy for persuasion?Question 3 options:Filter the EvidenceFlatter the audienceDivide the PressDivide the Opposition What does platano mean why cobra is called "king" of snake?